Đến nội dung

Hình ảnh

Tìm GTNN của A=$\prod (a^{2}+1)$

* - - - - 1 Bình chọn

  • Please log in to reply
Chủ đề này có 9 trả lời

#1
caokhanh97

caokhanh97

    Hạ sĩ

  • Thành viên
  • 78 Bài viết
Cho a,b,c,d không âm có tổng bằng 4 Tìm GTNN của A=$\prod (a^{2}+1)$
C.K

#2
khanh3570883

khanh3570883

    Trung úy

  • Thành viên
  • 905 Bài viết
Đã có trong diễn đàn!

Ta có:
$P = \left( {1 + {a^2}} \right)\left( {1 + {b^2}} \right)\left( {1 + {c^2}} \right)\left( {1 + {d^2}} \right) \le \left[ {\frac{{{{\left( {a + b} \right)}^2}}}{{16}} - \frac{{{{\left( {a + b} \right)}^2}}}{2} + 1} \right]\left[ {\frac{{{{\left( {c + d} \right)}^2}}}{{16}} - \frac{{{{\left( {c + d} \right)}^2}}}{2} + 1} \right]$
Đặt:
$\left\{ \begin{array}{l}
a + b - 2 = x \\
c + d + 2 = y \\
\end{array} \right. \Rightarrow x + y = 4$
Khi đó:
$P = \frac{{{{\left( {{x^2} - 4} \right)}^4}}}{{256}} - \frac{{{{\left( {{x^2} - 4} \right)}^2}\left[ {{x^2} + 4} \right]}}{{16}} + \frac{{{x^4} + 24{x^2} + 16}}{8} + \frac{{{{\left( {{x^2} - 4} \right)}^2}}}{4} - \frac{{{x^2} + 4}}{2} + 1$
Đến đây dùng đạo hàm tìm được max = 25


THẬT THÀ THẲNG THẮN THƯỜNG THUA THIỆT

LƯƠN LẸO LUỒN LỎI LẠI LEO LÊN

 

Một ngày nào đó ta sẽ trở lại và lợi hại hơn xưa


#3
WhjteShadow

WhjteShadow

    Thượng úy

  • Phó Quản lý Toán Ứng dụ
  • 1323 Bài viết

Đã có trong diễn đàn!

Nhưng ở đây là tìm GTNN mà anh Khánh.Bạn Khánh thử xem lại đề bài xem
“There is no way home, home is the way.” - Thich Nhat Hanh

#4
alex_hoang

alex_hoang

    Thượng úy

  • Hiệp sỹ
  • 1152 Bài viết

Cho a,b,c,d không âm có tổng bằng 4 Tìm GTNN của A=$\prod (a^{2}+1)$

Bài giải
Do $a+b+c+d=4$ nên tồn tại hai số cùng không lớn hơn $1$ hoặc cùng không nhỏ hơn $1$
Giả sử hai số đó là $b,d$ vậy thì $(b-1)(d-1) \ge 0$ suy ra $bd+1 \ge b+d$
Ta thấy
$$(a^2+1)(b^2+1)(c^2+1)(d^2+1)=(1+a^2+b^2+a^2b^2)(c^2+1+d^2+c^2d^2) \ge (c+a+bd+1)^2 \ge (a+b+c+d)^2=16$$
alex_hoang


HẸN NGÀY TRỞ LẠI VMF THÂN MẾN

http://www.scribd.co...oi-Ban-Cung-The

#5
Secrets In Inequalities VP

Secrets In Inequalities VP

    Sĩ quan

  • Thành viên
  • 309 Bài viết

Bài giải
Do $a+b+c+d=4$ nên tồn tại hai số cùng không lớn hơn $1$ hoặc cùng không nhỏ hơn $1$
Giả sử hai số đó là $b,d$ vậy thì $(b-1)(d-1) \ge 0$ suy ra $bd+1 \ge b+d$
Ta thấy
$$(a^2+1)(b^2+1)(c^2+1)(d^2+1)=(1+a^2+b^2+a^2b^2)(c^2+1+d^2+c^2d^2) \ge (c+a+bd+1)^2 \ge (a+b+c+d)^2=16$$

Em nghĩ có lẽ anh nhầm rồi ! Cách này chỉ đúng vz $abcd=1$ thôi ! :mellow:

#6
alex_hoang

alex_hoang

    Thượng úy

  • Hiệp sỹ
  • 1152 Bài viết

Em nghĩ có lẽ anh nhầm rồi ! Cách này chỉ đúng vz $abcd=1$ thôi ! :mellow:

Cho anh phản ví dụ cái nào em,trước khi nói là nhầm hoặc sai thì phải đưa ra phản ví dụ là anh chịu ngay.Theo anh cái này chỉ dựa vào nguyên lý $Dirichlet$ mà thôi.Có $4$ số mà chỉ có 2 TH là cùng không lớn hơn $1$ hoặc không nhở hơn $1$ thì chắc là được :closedeyes: .Anh nghĩ không nên vận dụng một cách máy móc một bài toán quá em à
alex_hoang


HẸN NGÀY TRỞ LẠI VMF THÂN MẾN

http://www.scribd.co...oi-Ban-Cung-The

#7
Secrets In Inequalities VP

Secrets In Inequalities VP

    Sĩ quan

  • Thành viên
  • 309 Bài viết

$$=(1+a^2+b^2+a^2b^2)(c^2+1+d^2+c^2d^2) \ge (c+a+bd+1)^2 \ge (a+b+c+d)^2=16$$

Ý e k phải vậy.anh giải thích hộ e đoạn này.Nếu k phải abcd=1 thì làm sao
có đk :(

Bài viết đã được chỉnh sửa nội dung bởi Secrets In Inequalities VP: 12-08-2012 - 17:24


#8
Tham Lang

Tham Lang

    Thượng úy

  • Thành viên
  • 1149 Bài viết
Anh cũng đồng ý với Secrets In Inequalities :D
@ Anh Hoàng : Có một chỗ đúng là nếu $abcd <1$ thì không thể có như vậy được
.

Bài giải
Do $a+b+c+d=4$ nên tồn tại hai số cùng không lớn hơn $1$ hoặc cùng không nhỏ hơn $1$
Giả sử hai số đó là $b,d$ vậy thì $(b-1)(d-1) \ge 0$ suy ra $bd+1 \ge b+d$
Ta thấy
$(a^2+1)(b^2+1)(c^2+1)(d^2+1)=(1+a^2+b^2+a^2b^2)(c^2+1+d^2+c^2d^2)$ $ \ge (c+a+bd+1)^2$ $ \ge (a+b+c+d)^2=16$


Bài viết đã được chỉnh sửa nội dung bởi Tham Lang: 12-08-2012 - 17:29

Off vĩnh viễn ! Không ngày trở lại.......


#9
alex_hoang

alex_hoang

    Thượng úy

  • Hiệp sỹ
  • 1152 Bài viết

Ý e k phải vậy.anh giải thích hộ e đoạn này.Nếu k phải abcd=1 thì làm sao
có đk :(

Anh cũng đồng ý với Secrets In Inequalities :D
@ Anh Hoàng : Có một chỗ đúng là nếu $abcd <1$ thì không thể có như vậy được
.

Cảm ơn các em nhé đúng là anh hơi chủ quan một chút.Mọi người thông cảm
Ta đặt $f(a,b,c,d)=(a^2+1)(b^2+1)(c^2+1)(d^2+1)$
Giả sử $a \ge b \ge c \ge d$
thì ta dễ chứng minh được
$$f(a,b,c,d) \ge f(\sqrt{ac},b,\sqrt{ac},d)$$
Như vậy ta chỉ cần chứng minh BĐT khi mà $a=b=c=t \ge 1$
Vậy ta chứng minh
$$f(t)=(t^2+1)^3((4-3t)^2+1) \ge 16$$ với $t \ge 1$
Nếu bài toán có $min$ thì BĐT trên phải đúng .Mình chưa thử các bạn xem giúp
alex_hoang


HẸN NGÀY TRỞ LẠI VMF THÂN MẾN

http://www.scribd.co...oi-Ban-Cung-The

#10
WhjteShadow

WhjteShadow

    Thượng úy

  • Phó Quản lý Toán Ứng dụ
  • 1323 Bài viết

Vậy ta chứng minh
$$f(t)=(t^2+1)^3((4-3t)^2+1) \ge 16$$ với $t \ge 1$

Điều cần chứng minh $\Leftrightarrow (t-1)^3(9t^5+3t^4+26t^3+6t^2+21t-1)\geq 0$
Nhưng điều này luôn đúng với $t\geq 1$
Vậy bất đẳng thức được chứng minh hoàn toàn anh Hoàng nhỷ :D

Bài viết đã được chỉnh sửa nội dung bởi WhjteShadow: 12-08-2012 - 20:32

“There is no way home, home is the way.” - Thich Nhat Hanh




0 người đang xem chủ đề

0 thành viên, 0 khách, 0 thành viên ẩn danh